2018 AMC 12B Problems/Problem 19

Revision as of 14:36, 16 February 2018 by Brainiac1 (talk | contribs) (Created page with "== Problem == Mary chose an even 4-digit number <math>n</math>. She wrote down all the divisors of <math>n</math> in increasing order from left to right: <math>1,2,..., \frac...")
(diff) ← Older revision | Latest revision (diff) | Newer revision → (diff)

Problem

Mary chose an even 4-digit number $n$. She wrote down all the divisors of $n$ in increasing order from left to right: $1,2,..., \frac{n}{2},n$. At some moment Mary wrote 323 as a divisor of $n$. What is the smallest possible value of the next divisor written to the right of 323?

\[\textbf{(A) } 324 \qquad \textbf{(B) } 330 \qquad \textbf{(C) } 340 \qquad \textbf{(D) } 361 \qquad \textbf{(E) } 646\]